2020-2021/TST/DM/2102_DM2/TST3/05_2102_DM2.tex

204 lines
7.5 KiB
TeX

\documentclass[a5paper,10pt]{article}
\usepackage{myXsim}
\usepackage{tasks}
% Title Page
\title{DM2 \hfill BOUCHOUX Kevin}
\tribe{TST}
\date{\hfillÀ render pour le Mercredi 24 février}
\xsimsetup{
solution/print = false
}
\begin{document}
\maketitle
\begin{exercise}[subtitle={Loi binomiale}]
Trois personnes s'apprêtent à passer le portique de sécurité. On suppose que pour chaque personne la probabilité que le portique sonne est égale à $0.7$.
Soit $X$ la variable aléatoire donnant le nombre de personnes faisant sonner le portique, parmi les 3 personnes de ce groupe.
\begin{enumerate}
\item Tracer l'arbre représentant le situation.
\item Justifier que $X$ suit une loi binomiale dont on précisera les paramètres.
\item Quelle est la probabilité qu'une seule personne fasse sonner le portique?
\item Calculer puis interpréter les probabilités suivantes
\[
P(X = 0) \qquad \qquad P(X \geq 2)
\]
\item Calculer l'espérance de $X$ et interpréter le résultat.
\end{enumerate}
\end{exercise}
\begin{solution}
\begin{enumerate}
\item
\begin{tikzpicture}[sloped]
\node {.}
child {node {$0$}
child {node {$0$}
child {node {$0$}
edge from parent
node[above] {0.3}
}
child {node {$1$}
edge from parent
node[above] {0.7}
}
edge from parent
node[above] {0.3}
}
child[missing] {}
child {node {$1$}
child {node {$0$}
edge from parent
node[above] {0.3}
}
child {node {$1$}
edge from parent
node[above] {0.7}
}
edge from parent
node[above] {0.3}
}
edge from parent
node[above] {0.3}
}
child[missing] {}
child[missing] {}
child[missing] {}
child { node {$1$}
child {node {$0$}
child {node {$0$}
edge from parent
node[above] {0.3}
}
child {node {$1$}
edge from parent
node[above] {0.7}
}
edge from parent
node[above] {0.3}
}
child[missing] {}
child {node {$1$}
child {node {$0$}
edge from parent
node[above] {0.3}
}
child {node {$1$}
edge from parent
node[above] {0.7}
}
edge from parent
node[above] {0.3}
}
edge from parent
node[above] {0.7}
} ;
\end{tikzpicture}
\item Chaque personne a 2 possibilités (1: fait sonner ou 2: ne fait pas sonner) et l'on fait passer 3 personnes ce qui correspond à une répétition identique et aléatoire. On peut donc modéliser la situation par une loi binomiale.
\[
X \sim \mathcal{B}(3; 0.76)
\]
\item Probabilité qu'une seule personne fasse sonner le portique. On voit qu'il y a 3 branches qui correspondent à cette situation dont
\[
P(X = 1) = 3 \times 0.7^1 \times 0.3^2 \approx 0.189
\]
\item
\[
P(X = 0) = 0.3^3 \approx 0.027
\]
\[
P(X \geq 2) = P(X = 2) + P(X = 3) = 3 \times 0.7^2 \times 0.3^1 + 0.7^3 \approx 0.784
\]
\item Il faut d'abord tracer le tableau résumant la loi de probabilité:
\begin{center}
\begin{tabular}{|c|*{4}{c|}}
\hline
Valeur & 0 & 1 & 2 & 3 \\
\hline
Probabilité & $0.027$ & $0.189$ & $0.441$ &$0.343$ \\
\hline
\end{tabular}
\end{center}
On peut alors calculer l'espérance
\[
E[X] = 0 \times 0.027 + 1 \times 0.189 + 2 \times 0.441 + 3 \times 0.343 = 2.1
\]
On peut donc estimer qu'il y aura en moyenne $2.1$ personnes qui feront sonner le portique sur les 3 personnes.
\end{enumerate}
\end{solution}
\begin{exercise}[subtitle={Équation puissance}]
Résoudre les équations et inéquations suivantes
\begin{multicols}{2}
\begin{enumerate}
\item $10^x = 4$
\item $7^x = 14$
\item $0.44^x \leq 29$
\item $6 \times 0.27^x = 10$
\end{enumerate}
\end{multicols}
\end{exercise}
\begin{solution}
Les solutions ci-dessous ne sont pas justifiée car l'ordinateur ne sait pas faire. Par contre, vous vous devez savoir justifier vos réponses!
\begin{enumerate}
\item $x = \log(4)$
\item $x = \frac{\log(14)}{\log(7)}$
\item Il faut faire attention quand on divise par un log car ce dernier peut être négatif ce qui est le cas ici. Il faut donc pense à changer le sens de l'inégalité.
$x \geq \frac{\log(29)}{\log(0.44)}$
\item Il faut penser à faire la division à par $6$ avant d'utiliser le log car sinon, on ne peut pas utiliser la formule $\log(a^n) = n\times \log(a)$.
$x = \frac{\log(1.67)}{\log(0.27)}$
\end{enumerate}
\end{solution}
\begin{exercise}[subtitle={Étude de fonctions}]
Soit $f(x) = 10x^3 - 840x^2 + 18450x - 1$ une fonction définie sur $\R$.
\begin{enumerate}
\item Calculer $f'(x)$ la dérivée de $f(x)$.
\item Calculer $f'(41)$ et $f'(15)$.
\item En déduire une forme factorisée de $f'(x)$.
\item Étudier le signe de $f'(x)$ et en déduire les variations de $f(x)$.
\item Est-ce que la fonction $f(x)$ admet un maximum ou un minimum? Si oui, calculer sa valeur.
\end{enumerate}
\end{exercise}
\begin{solution}
\begin{enumerate}
\item Dérivée de $f(x)$: $f'(x) = 30x^2 - 1680x + 18450$
\item
\begin{align*}
f'(41) &= 30 \times 41^{2} - 1680 \times 41 + 18450\\&= 30 \times 1681 - 68880 + 18450\\&= 50430 - 50430\\&= 0
\end{align*}
\begin{align*}
f'(15) &= 30 \times 15^{2} - 1680 \times 15 + 18450\\&= 30 \times 225 - 25200 + 18450\\&= 6750 - 6750\\&= 0
\end{align*}
Donc $x = 41$ et $x=15$ sont des racines de $f'(x) = 30x^2 - 1680x + 18450$.
\item On en déduit la forme factorisée suivante
\[
f'(x) = 30 (x - 41)(x-15)
\]
\item Pas de correction disponible
\item À causes des branches extérieurs, la fonction $f(x)$ n'a pas de maximum ou de minimum.
\end{enumerate}
\end{solution}
%\printsolutionstype{exercise}
\end{document}
%%% Local Variables:
%%% mode: latex
%%% TeX-master: "master"
%%% End: